Câu hỏi:
2 năm trước

Cho hàm số \(f\left( x \right) = \left\{ \begin{array}{l}2x + 3\,\,\,\,\,\,\,\,\,\,\,\,\,\,\,\,\,\,\,\,\,\,\,\,\,khi\,\,x \ge 1\\\dfrac{{{x^3} + 2{x^2} - 7x + 4}}{{x - 1}}\,\,khi\,\,x < 1\end{array} \right.\). Giá trị của \(f'\left( 1 \right)\) bằng:

Trả lời bởi giáo viên

Đáp án đúng: d

$\mathop {\lim }\limits_{x \to {1^ + }} \dfrac{{f\left( x \right) - f\left( 1 \right)}}{{x - 1}} $ $= \mathop {\lim }\limits_{x \to {1^ + }} \dfrac{{2x + 3 - 5}}{{x - 1}} $ $= \mathop {\lim }\limits_{x \to {1^ + }} \dfrac{{2x - 2}}{{x - 1}} = 2$

$\mathop {\lim }\limits_{x \to {1^ - }} \dfrac{{f\left( x \right) - f\left( 1 \right)}}{{x - 1}} $ $= \mathop {\lim }\limits_{x \to {1^ - }} \dfrac{{\dfrac{{{x^3} + 2{x^2} - 7x + 4}}{{x - 1}} - 5}}{{x - 1}}$

$ = \mathop {\lim }\limits_{x \to {1^ - }} \dfrac{{{x^3} + 2{x^2} - 12x + 9}}{{{{\left( {x - 1} \right)}^2}}} $ $= \mathop {\lim }\limits_{x \to {1^ - }} \dfrac{{\left( {x - 1} \right)\left( {{x^2} + 3x - 9} \right)}}{{{{\left( {x - 1} \right)}^2}}} $ $= \mathop {\lim }\limits_{x \to {1^ - }} \dfrac{{{x^2} + 3x - 9}}{{x - 1}} =  + \infty $

$ \Rightarrow \mathop {\lim }\limits_{x \to {1^ + }} \dfrac{{f\left( x \right) - f\left( 1 \right)}}{{x - 1}} \ne \mathop {\lim }\limits_{x \to {1^ - }} \dfrac{{f\left( x \right) - f\left( 1 \right)}}{{x - 1}}$

Vậy hàm số không tồn tại đạo hàm tại $x = 1.$

Hướng dẫn giải:

Đạo hàm của hàm số \(y = f\left( x \right)\) tại điểm \(x = {x_0}\) là \(f'\left( {{x_0}} \right) = \mathop {\lim }\limits_{x \to {x_0}} \dfrac{{f\left( x \right) - f\left( {{x_0}} \right)}}{{x - {x_0}}}\) (nếu tồn tại).

Câu hỏi khác